2
$\begingroup$

Let $k$ be a field. For example, $k=\mathbb{Q}$ or $\mathbb{Z}/p$, $p$ prime.

Let $k[x,y]$ be the polynomial ring.

Let $f,g\in k[x,y]$.

Let the ideal $I=(f,g)$ be the ideal of $k[x,y]$ generqated by $f,g$.

Question. For $h\in k[x,y]$, I want to find whether there exists a smallest integer $n$ such that $h^n\in I$.

Is there any method?

Also I want to compute out the explicit smallest integer $n$.

$\endgroup$

2 Answers 2

3
$\begingroup$

If you want to determine whether there is an $n$ such that $h^n\in I$, consider the following (this requires passing to the algebraic closure):

$h^n\in I$ for some $I$ if and only if $h\in\sqrt{I}$ where $\sqrt{I}$ is the radical of $I$.

This is the same as $1\in\langle f,g,1-zh\rangle$ where $z$ is a new variable. This follows from the following:

Suppose (for contradiction) that $p\in\mathcal{V}(\langle f,g,1-zh\rangle)$ and $h\in\sqrt{I}$. Then, $f(p)=0$ and $g(p)=0$, so $h(p)=0$ since $h^n=af+bg$ and $h^n(p)=af(p)+bg(p)=0$. Therefore, $1-zh(p)\not=0$, a contradiction. Therefore, $\mathcal{V}(\langle f,g,1-zh\rangle)=\emptyset$, and, by the Nullstellensatz, $1\in\langle f,g,1-zh\rangle$.

On the other hand, if $1\in\langle f,g,1-zh\rangle$, then, at any point $p\in\mathcal{V}(\langle f,g\rangle)$, $h(p)$ is zero because, otherwise, $(p,1/h(p))$ would be in $\mathcal{V}(\langle f,g,1-zh\rangle)$. Therefore, $h$ vanishes on $\mathcal{V}(\langle f,g\rangle)$ and (again by the Nullstellensatz), $h$ is in the radical of $I$.

To summarize, you must compute a Groebner basis for $\langle f,g,1-zh\rangle$ and determine whether $1$ is in that ideal.

$\endgroup$
0
$\begingroup$

The magic words are Grobner basis and "saturation" (search for it in the wiki page).

$\endgroup$
1
  • $\begingroup$ It's not so clear if the OP wants to compute the $n$ or to just find out whether some power of $h$ is in $I$. $\endgroup$ Apr 17, 2015 at 3:19

Your Answer

By clicking “Post Your Answer”, you agree to our terms of service and acknowledge you have read our privacy policy.

Not the answer you're looking for? Browse other questions tagged or ask your own question.